Basic skills modules (MCQ) Flashcards

(158 cards)

1
Q

A 70 year old woman with acute pulmonary edema has a respiratory rate of 30/min, heart rate 120/min, BP 170/90 and SpO2 91% despite oxygen 15L/min via a reservoir facemask, frusemide IV and IV glyceryl tri-nitrate infusion. On auscultation of her chest she has bilateral crackles. What would you do? (P.80)
Select one or more:
a. Intubate and ventilate
b. Start continuous positive airway pressure non-invasive ventilation
c. Start bilevel airway pressure non-invasive ventilation
d. Increase dose of glyceryl trinitrate
e. IV morphine

A

B,E
Explanation: The patient has cardiogenic pulmonary oedema with moderately severe hypoxia despite high flow oxygen and hypertension. You need to increase her supportive therapy and maximize the treatment of her heart failure. CPAP should be tried before BiPAP for cardiogenic pulmonary oedema. Her hypertension is probably due to high sympathetic drive due to the discomfort of pulmonary oedema. It will worsen her heart failure due to high afterload. IV morphine will reduce the discomfort. GTN will reduce her preload and, to a lesser extent, the afterload. The major contraindication is hypotension, but she is not hypotensive. It is very short acting so its effect will disappear rapidly if the patient’s BP falls after giving morphine.

How well did you know this?
1
Not at all
2
3
4
5
Perfectly
2
Q

A patient has suddenly become unconscious in his bed, and you are the first doctor to arrive. The patient is not breathing. His pulse is present, and regular. The nurse hands you a bag-mask resuscitator and you attempt to ventilate the patient after applying the head tilt, chin lift procedure. The chest fails to rise when the bag is squeezed. Your immediate airway management should be:
Select one:
a. Triple manoeuvre
b. Check and ensure adequate seal around the mask
c. Intubation
d. Two-person technique to assist ventilation
e. Placement of an oropharyngeal airway

A

B

How well did you know this?
1
Not at all
2
3
4
5
Perfectly
3
Q

You see an elderly man collapse on the grass in a park. The environment is safe. He is unresponsive. Your next step should be to:

A

Call for help

How well did you know this?
1
Not at all
2
3
4
5
Perfectly
4
Q

The most effective way to prevent harm to patients is to eliminate mistakes
1. True
2. False

A

2

How well did you know this?
1
Not at all
2
3
4
5
Perfectly
5
Q

These are the results from a 25 year old 70 kg patient with pneumonia: BP 100/50 (mean 65) mmHg, JVP 0 cm above sternal angle, urine output 50 ml/hr, pH 7.4. The patient is alert and has warm peripheries. The most appropriate haemodynamic intervention is:
Select one:
a. IV bolus of 250ml 5% glucose
b. IV bolus of 500ml crystalloid
c. Dopamine infusion
d. No intervention is required
e. Dobutamine infusion

A

D

How well did you know this?
1
Not at all
2
3
4
5
Perfectly
6
Q

50 year old patient with ischaemic chest pain is breathing on a Hudson mask (wall oxygen 10L/min). If his respiratory system is normal his expected PaO2 on ABG would be [similar questions]
Select one:
a. 10kPa
b. 20kPa
c. 45kPa
d. 90kPa
e. 95kPa

A

C

How well did you know this?
1
Not at all
2
3
4
5
Perfectly
7
Q

50 year old patient with ischaemic chest pain is breathing on a Hudson mask (wall oxygen 6L/min). If his respiratory system is normal his expected PaO2 on ABG would be [similar questions]
Select one:
a. 50kPa
b. 95kPa
c. 10kPa
d. 70kPa
e. 30 kPa

A

E

(Around FIO2 * 95 - 6)

Let’s analyze this scenario step-by-step:

  • The patient is on a Hudson mask delivering oxygen at 6L/min. Typically, a Hudson mask (also known as a simple mask) provides a FiO₂ (fraction of inspired oxygen) approximately between 40-60%, depending on the flow rate and patient breathing.
  • At 6L/min, the approximate FiO₂ is around 40-50%. For simplicity, we’ll consider it roughly 40%.
  • Normal alveolar oxygen partial pressure (PAO₂) can be estimated using the alveolar gas equation:[
    PAO_2 = (FiO_2 \times (Patm - PH_2O)) - \frac{PaCO_2}{RQ}
    ]where:
    - ( Patm ) (atmospheric pressure) ≈ 101 kPa at sea level
    - ( PH_2O ) (water vapor pressure) ≈ 6.3 kPa
    - ( RQ ) (respiratory quotient) ≈ 0.8
    - ( PaCO_2 ) (arterial CO₂) is usually around 5.3 kPa in a normal respiratory system.
  • Plugging in the values:[
    PAO_2 = 0.40 \times (101 - 6.3) - \frac{5.3}{0.8}
    ][
    PAO_2 = 0.40 \times 94.7 - 6.625
    ][
    PAO_2 ≈ 37.88 - 6.63 ≈ 31.25\, \text{kPa}
    ]
  • In a normal respiratory system, the arterial oxygen tension (PaO₂) is typically slightly less than PAO₂ due to normal physiologic shunting (~5%), so:[
    PaO_2 \approx PAO_2 - 5 \text{ kPa} \approx 26-30\, \text{kPa}
    ]
  • Since the question asks for the expected PaO₂ in a normal respiratory system, the value should be approximately 30 kPa.

Therefore, the correct answer is:

e. 30 kPa

How well did you know this?
1
Not at all
2
3
4
5
Perfectly
8
Q

You are resuscitating a patient that you believe to have septic shock. So far he has received 3l of Hartmann’s solution. His BP rose on passive leg raising from 70/50 to 80/50 when you checked 1 minute ago. You now notice that he has stopped talking. He extends to pain, eye opens to pain, and makes no sounds. BP which was 70/40 has now fallen to 50/30. The next management step is:
Select one:
a. IV epinephrine 0.1mg
b. Insert a second large IV cannula and start a colloid infusion
c. Call ICU/anaesthetist to intubate patient
d. Start an infusion of dopamine at 5mcg/kg/min
e. Insert a second large IV cannula and increase the rate of infusion of Hartmann’s solution

A

A

How well did you know this?
1
Not at all
2
3
4
5
Perfectly
9
Q

You are resuscitating a patient that you believe to have septic shock. So far he has received 3l of Hartmann’s solution. His BP rose on passive leg raising from 90/50 to 95/55 when you checked 1 minute ago. You now notice that he has stopped talking. He localizes to pain, eye opens to speech, and makes incomprehensible sounds. BP which was 90/50 has now fallen to 80/40. The immediate management is:
Select one:
a. Insert a second large IV cannula and increase the rate of infusion of Hartmann’s solution
b. Call ICU/anaesthetist to intubate patient
c. Start an infusion of dopamine at 5 mcg/kg/min
d. Obtain norepinephrine from ICU and start an infusion at 5 mcg/min
e. IV epinephrine 1 mg

A

C

D: possibly not obtaining from an ICU — require immediate infusion

How well did you know this?
1
Not at all
2
3
4
5
Perfectly
10
Q

A 70 year old diabetic patient is admitted with fever, right upper quadrant abdominal pain and vomiting of undigested food. His blood pressure is 90/40mmHg, pulse 122/minute, temperature 39.6 °C, SpO2 95% on room air. He is jaundiced and he has tenderness over the right upper quadrant. Which of the following investigations should take place next? (P. 95)
[similar questions]
Select one:
a. Arterial blood gas
b. Two sets of blood cultures
c. Passive leg raising
d. Ultrasound of the haptobiliary system
e. CT abdomen

A

C

How well did you know this?
1
Not at all
2
3
4
5
Perfectly
11
Q

A 70 year old diabetic patient is admitted with fever, right upper quadrant abdominal pain and vomiting of undigested food. His blood pressure is 90/40mmHg, pulse 102/minute , temperature 39.6 °C. He is jaundiced and he has tenderness over the right upper quadrant. Which of the following investigations should take place next? (P. 96)
[similar questions]
Select one:
a. ultrasound of the hepatobiliary system
b. MRCP
c. ERCPintrq
d. Two sets of blood cultures
e. CT abdomen

A

D

How well did you know this?
1
Not at all
2
3
4
5
Perfectly
12
Q

(lack explanations)

Rank the following patients in terms of the urgency with which they require treatment, from most urgent to least.
Patient X
Elderly patient with pneumonia. Decreased consciousness, respiratory rate 6 breaths per minute, heart rate 60 beats per minute, airway patent.
Patient Y
Elderly patient with urinary tract infection, BP 80/50, heart rate 124 per minute, drowsy, oliguric.
Patient Z
Elderly patient with decreased consciousness following a fit, GCS 6, airway patent, capillary glucose 4 mmol/l, recovery position

A

X,Y,Z

How well did you know this?
1
Not at all
2
3
4
5
Perfectly
13
Q

While you are doing blood culture for 50 years old man in ward, you find the other elderly patient next to your patient collapses. Your next action should be
Select one:
a. Shout for help
b. Assess his pulse
c. Assess his airway
d. Call cardiac arrest team
e. Assess his breathing

A

A

How well did you know this?
1
Not at all
2
3
4
5
Perfectly
14
Q

A 59-year-old man has undergone a prostatectomy and has suffered the complication of urinary sepsis (he has E. coli positive culture in the urine and in blood culture). He has received 2.5 L of 2.5% dextrose and 0.45% saline fluid infusion over the last 24 hours. Previously awake and alert, the nurse now calls you to the bedside and tells you the following:
He has become confused and drowsy.
The nurse has placed a non-rebreathing mask on the patient, which is misting up with each exhalation.
His arterial oxygen saturation is 98%
His BP was 80/60mmHg just before she called you
You repeat the BP reading and simultaneously check his JVP, which is visible about 1cm above the sternal angle.
The BP is now 46/27mmHg.
After calling for help, your first action should be:
Select one:
a. Insert an oropharyngeal tube to keep the airway open
b. Immediately order 500mL of normal saline to be given intravenously as fast as possible
c. Administer intravenous epinephrine 0.1mg IV immediately
d. Immediately prepare for intubation and transfer to ICU
e. Immediately order 100ml normal saline to be given intravenously as fast as possible

A

C

How well did you know this?
1
Not at all
2
3
4
5
Perfectly
15
Q

A 63-year-old man has recently returned to Hong Kong from a trip to the United States of America. He presents to A&E with a sudden onset of chest discomfort and severe shortness of breath. He is shocked and hypoxic. The chest X-ray is clear. Which of the following signs or symptoms best match his diagnosis.
Select one or more:
a. Jugular venous pressure (JVP), patient sitting at 45 degrees, visible 5cm above the level of sternal notch
b. Jugular venous pressure (JVP), patient sitting at 45 degrees, just visible above the level of the sternal notch
c. Chest pain like a knife cutting into me, worse when I cough
d. Chest pain like a vice, crushing my chest
e. Burning chest pain, worse when laying down, or sitting watching TV
f. Cold peripheries
g. Warm peripheries
h. Capillary refill time of 4s
i. Capillary refill time of 2s
j. Lactate of 5 mmol/L
k. Lactate of 1 mmol/L
l. Urine output of 200 ml over 12 hours
m. Urine output of 200 ml over 3 hours
n. Swollen, red and warm calf (unilateral)
o. Bi-basal, fine, inspiratory crackles
p. Unilateral, basal, fine, inspiratory crackles
q. Bronchial breathing and coarse crackles
r. Bi-basal, decreased breath sounds
s. Bilateral expiratory wheezes
t. Inspiratory stridor

A

a,f,h,j,n

a. Jugular venous pressure (JVP), patient sitting at 45 degrees, visible 5cm above the level of sternal notch

Indicates significant jugular venous distension (JVD), suggestive of right heart strain or failure such as in massive PE.
This is consistent with PE causing right ventricular overload.
b. Jugular venous pressure (JVP), patient sitting at 45 degrees, just visible above the level of the sternal notch

Less prominent, less specific but still could be related.
d. Chest pain like a vice, crushing my chest

Classic description of myocardial infarction, but in PE, chest pain can also be pleuritic and sharp.
Less typical for PE but possible.
e. Burning chest pain, worse when laying down, or sitting watching TV

Describes reflux or angina, less specific here.
g. Warm peripheries

Indicates adequate perfusion, less likely in shock.
f. Cold peripheries

Indicative of shock, especially hypoperfusion, which matches this patient.
j. Lactate of 5 mmol/L

Elevated lactate indicates tissue hypoxia and shock.
k. Lactate of 1 mmol/L

Normal, less likely in shock.
l. Urine output of 200 ml over 12 hours

Very low (oliguria), indicating hypoperfusion.
m. Urine output of 200 ml over 3 hours

Still low, but less than 12 hours, indicates shock.
n. Swollen, red, and warm calf (unilateral)

Classic for deep vein thrombosis (DVT), which can cause PE.
o. Bi-basal, fine, inspiratory crackles

Common in pulmonary edema, less specific here.
p. Unilateral, basal, fine, inspiratory crackles

Suggests pneumonia or localized pathology.
q. Bronchial breathing and coarse crackles

Less typical for PE.
r. Bi-basal, decreased breath sounds

Could indicate pleural effusion or collapse.
s. Bilateral expiratory wheezes

Asthma or airway obstruction.
t. Inspiratory stridor

Upper airway obstruction, unlikely here.

How well did you know this?
1
Not at all
2
3
4
5
Perfectly
16
Q

An obese 55 year old man with body weight of 120kg is admitted to the ward for spontaneous intracranial haemorrhage. His GCS has deteriorated from 14/15 to 3/15 and you are called to assist in managing this patient. His respiratory rate is 12 /min. After giving him 15L/min Oxygen via non-rebreathing mask, his oxygen saturation is 85%. What is your next management?
Select one:
a. Do a head tilt chin lift
b. Perform bag valve mask ventilation
c. Inform your senior and observe
d. Proceed to direct laryngoscopy and intubation
e. Proceed to fibre-optic intubation

A

A

How well did you know this?
1
Not at all
2
3
4
5
Perfectly
17
Q

(Lack explanations)

A 64 year old man presents with sudden onset of shortness of breath over minutes. He has a history of poorly controlled hypertension and angina. On examination:
Heart rate 130/min, sinus tachycardia Blood pressure 90/60
JVP 7 cm above sternal angle
Gallop rhythm, no murmur
Cold peripheries
Respiratory rate 35/min
Bilateral crackles
SpO2 89% on 15 l/min oxygen via reservoir facemask
Appropriate respiratory support is: (p.81)
Select one or more:
a. BiPAP
b. Refer to ICU for invasive ventilation
c. No change is necessary
d. CPAP
e. 60% oxygen by facemask

A

B

How well did you know this?
1
Not at all
2
3
4
5
Perfectly
18
Q

Dual antibiotic therapy is recommended for: Select one:
a. Meningococcal meningitis
b. Penicillin sensitive Streptococcus pneumoniae pneumonia
c. Enterobacter pneumonia
d. Methicillin resistant Staphylococcus aureus pneumonia
e. E. Coli bacteraemia

A

C

How well did you know this?
1
Not at all
2
3
4
5
Perfectly
19
Q

A 50 year old man with a history of gout presents to your GP clinic. He hurt his ankle 3 days ago while playing football with his sons. Initially the pain in the ankle improved but over the past day it has become more painful, hot and swollen. He has developed diarrhoea and vomiting and his wife says he is intermittently confused.
His management should include: Select one or more:
a. Drainage of the joint
b. Anti-psychotic
c. Oral antibiotics
d. Allopurinol
e. Urgent referral to hospital
f. IV antibiotics

A

A,E,F

How well did you know this?
1
Not at all
2
3
4
5
Perfectly
20
Q

A 23 year old, 60 kg asthmatic presents with increasing shortness of breath and wheeze over 1 day. On examination she is alert and able to complete full sentences, is not using her accessory muscles of respiration and has a respiratory rate of 22 breaths per min. Her heart rate is 98/min, pulsus paradoxus 5 mmHg, PEFR 220 l/min. She has received no treatment apart from 2 puffs of a salbutamol inhaler 4 hourly. Appropriate management would be: (P. 40)
A. Select one:
treat with inhaled bronchodilators and discharge home
B. treat with inhaled bronchodilators and intravenous steroids and admit to medical ward for observation
C. treat with inhaled bronchodilators and oral steroids and discharge home
D. treat with inhaled bronchodilators and intravenous steroids and refer to ICU for
ventilation
E. treat with intravenous bronchodilators and intravenous steroids and admit to medical
ward for observation

A

B

How well did you know this?
1
Not at all
2
3
4
5
Perfectly
21
Q

The correct technique of chest compressions is (choose all correct answers):
Select one or more:
A. Compression-ventilation ratio should be 30:2 until the patient is intubated
B. The depth of compression should be at least 6 cm
C. The chest compression rate should be 100-120/min
D. The heel of your hand should be placed just to the left of the sternum

A

A,C

How well did you know this?
1
Not at all
2
3
4
5
Perfectly
22
Q

A 65 year old man with bleeding oesophageal varices develops hypotension: BP 50/30, HR 130/min, JVP 0, cold peripheries. He is vomiting blood. His airway is patent but he has become unconscious. His pulse oximetry is undetectable but was previously 97% on air.
Your immediate action should be: Select one: (P.57)
A. Give IM epinephrine 0.5mg
B. Give IV epinephrine 1mg
C. Give IV epinephrine 0.1mg
D. Give blood as fast as possible
E. Call for an aanesthetist to intubate the patient

A

C

How well did you know this?
1
Not at all
2
3
4
5
Perfectly
23
Q

A 40 years old lady presents to medical ward because of shortness of breath. She is found to have bilateral lower zone crepitations, a loud first heart sound and a localized diastolic murmur at the apex. Her CXR is as follows,
What is the most likely cause of her symptoms? Select one:
A. Viral pneumonia
B. Mitral stenosis
C. Pulmonary tuberculosis
D. Myocardial ischaemia
E. Bacterial pneumonia

A

B

How well did you know this?
1
Not at all
2
3
4
5
Perfectly
24
Q

You have been informed about four patients. Which order would you see them in?

Patient A
54 year old woman admitted from the emergency department with breathlessness. Her arterial blood gas on room air shows pH 7.1, PaCO2 1.5 kPa (11 mmHg), PaO2 9 kPa (68 mmHg). The nurse has already started giving oxygen 6 l/min via facemask.
Patient B
72 year old man with 10 minutes of central crushing chest pain which resolved with sublingual GTN.
Patient C
84 year old woman with pneumonia. Her respiratory rate is 30/min, SpO2 94% on 15L/min oxygen via reservoir facemask, BP 110/40, urine output 20 ml over past hour. The nurse asks you to see her because she is confused and agitated.
Patient D
65 year old man with a history of an abdominal aortic aneurysm who has BP 60/40 and heart rate 130/min

A

D,A,C,B

(How abt DCAB?)

How well did you know this?
1
Not at all
2
3
4
5
Perfectly
25
A 22 year old girl with known asthma was admitted for progressive shortness of breath associated with fever and productive cough for 2 days. She was given Augmentin and salbutamol inhaler by her family doctor. Her blood pressure is now 100/50mmHg, pulse is regular at 120 beats/minute. Her SpO2 is 85 % on 10 L/min oxygen via Hudson mask and respiratory rate is 30 breaths/minute. On examination of the lungs, there is bilateral expiratory wheeze and left lower zone bronchial breath sounds with percussion dullness. ABG shows pH 7.2 pCO2 10.0 pO2 7.5 HCO3 18 BE -5. Adequate dose salbutamol and ipratropium inhalers has already been started. What should be the immediate next step of your management? Select one: a. Give 15L/min O2 via reservoir facemask b. Perform a chest X-ray c. Consult ICU d. Give hydrocortisone intravenously e. Save cultures and give IV antibiotics
A B: (not this, This is very severe asthma and the patient requires urgent referral to ICU. Should you do anything in the meantime?) C: (partially correct, This is very severe asthma and the patient requires urgent referral to ICU. Should you do anything in the meantime?)
26
You assess a 50 years old woman in shock. Her blood pressure is 80/50 with pulse rate of 110/min and warm periphery. Which of the following suggest she will respond to intravenous fluid? Select one: a. Cold skin over distal limbs b. A rise of systolic pressure by 10% on passive leg raising c. A rise of central venous pressure of 7mmHg after fluid bolus d. A rise of central venous pressure of 2mmHg after fluid bolus e. A rise of diastolic pressure by 10% on passive leg raising
D (Why not B? And even if it’s D why not C?)
27
A 45 year old man is admitted to the medical ward because of fever and bloody diarrhoea. He complains of thirst and looks tired after admission. He is found to have high fever, decreased skin turgor and tachycardia after admission. 3L of saline has been given in the past 2 hours. His current BP changes from 82/35 to 83/36 after passive leg raising. You can feel his peripheries are warm. What is your next step of management in the ward? Select one: a. Give intravenous adrenaline 0.1mg b. Start dobutamine infusion c. Start dopamine infusion d. Give one unit of pack cell full rate e. Give saline 500ml full rate
C
28
You have witnessed a man suddenly collapse in the street. He is pulseless, but is gasping. There are no safety hazards. You cannot see anyone nearby. The correct immediate management (in order of priority) is:
Shout for help -> phone for an ambulance
29
A 86 year old man who underwent a hemicolectomy 5 days ago had a blood pressure of 70/40, warm peripheries and a normal jugular venous pressure. He has been fluid resuscitated with 2L normal saline and is now haemodynamically stable but remains drowsy and confused with some abdominal tenderness. Which of the following investigations would be your main priority, in addition to routine biochemistry, haematology and arterial blood gas: Select one or more: a. CT abdomen b. 12 lead ECG c. Echocardiography d. Chest X-ray e. CT brain and lumbar puncture
A,D
30
A 65 year old male smoker with coronary artery disease presents to the Accident & Emergency department with sudden onset of shortness of breath coming on over a few minutes. The most likely cause of his dyspnoea is: (P. 79) Select one: a. Exacerbation of chronic obstructive airways disease b. Metabolic acidosis c. Acute pulmonary oedema d. Asthma e. Pneumonia
C
31
You have been asked to see 4 patients. Which order would you see them in? [version 1] 62 year old man admitted with chest pain and T wave inversion is his inferior ECG leads. He has already been started on aspirin. The nurse rings you to tell you that his troponin concentration is raised. His heart rate is 75/min, BP 140/80, SpO2 95% on air, respiratory rate 15/min and his chest pain has resolved. 65 year old, 60 kg patient who underwent an elective total colectomy 4 days ago. You have been called because his urine output has been 20 ml/h for the past 2 hours and he is confused. His heart rate is 110/min, RR 28/min, SpO2 96% on O2 4 l/min via nasal prongs. 32 year old asthmatic who was admitted to a medical ward 10 hours ago. She has been treated with a salbutamol inhaler 4 puffs hourly, ipratropium inhaler 4 puffs 6 hourly and IV hydrocortisone 200 mg 8 hourly. Her peak flow has fallen from 120 l/min on admission to 60 l/min and she has developed a respiratory acidosis (pH 7.19). Her respiratory rate is 35/min, heart rate 120/min, SpO2 100% on 4 l/min oxygen via a simple oxygen mask. 55 year old, known epileptic, admitted for an elective cholecystectomy tomorrow. He has just had a generalized convulsion lasting for 1 minute and stopping spontaneously. His GCS is 7 but his airway is patent, he is breathing at 15 breaths/min, SpO2 100% on oxygen 2L/min via nasal prongs, BP 130/70, heart rate 90/min, glucose 5 mmol/l. The nurses have already turned him to the recovery position
C,B,D,A
32
You have been asked to see 4 patients. Which order would you see them in? [version 2] 32 year old asthmatic who was admitted to a medical ward 10 hours ago. She has been treated with a salbutamol inhaler 4 puffs hourly, ipratropium inhaler 4 puffs 6 hourly and IV hydrocortisone 200 mg 8 hourly. Her peak flow has risen from 120 l/min on admission to 180 l/min. Her PaCO2 is 5 kPa (38 mmHg). Her respiratory rate is 25/min, heart rate 110/min, SpO2 100% on 4 l/min oxygen via a simple oxygen mask. 62 year old man admitted with chest pain and T wave inversion is his inferior ECG leads. He has already been started on aspirin. The nurse rings you to tell you that his troponin concentration is raised. His heart rate is 75/min, BP 140/80, SpO2 95% on air, respiratory rate 15/min and his chest pain has resolved. 65 year old, 60 kg patient who underwent an elective total colectomy 4 days ago. You have been called because his urine output has been 20 ml/h for the past 2 hours and he is confused. His heart rate is 110/min, RR 38/min, SpO2 96% on O2 4 l/min via nasal prongs. 55 year old, admitted for an elective cholecystectomy tomorrow. He had a generalized convulsion lasting for 1 minute which stopped spontaneously. His GCS recovered to 7 after his convulsion but is now convulsing again. The nurses report that his airway is patent, he is breathing and they have already turned him to the recovery position.
D,C,A,B (D: active seizure -> need urgent management)
33
You have been asked to see 4 patients. Which order would you see them in? [version 3] 32 year old asthmatic who was admitted to a medical ward 10 hours ago. She has been treated with a salbutamol inhaler 4 puffs hourly, ipratropium inhaler 4 puffs 6 hourly and IV hydrocortisone 200 mg 8 hourly. Her peak flow has fallen from 120 l/min on admission to 80 l/min in the past 4 hours. Her PaCO2 is 5 kPa (38 mmHg). Her respiratory rate is 30/min, heart rate 120/min, SpO2 100% on 4 l/min oxygen via a simple oxygen mask. 62 year old man admitted with chest pain and ST elevation in the anterior ECG leads. He has already been started on aspirin and has received thrombolysis. The nurse rings you to tell you that his heart rate is 125/min, BP 70/40, SpO2 92% on 15 l/min oxygen through a non-rebreathing mask, respiratory rate 10/min and he is drowsy. 65 year old, 60 kg patient who underwent an elective total colectomy 4 days ago. You have been called because he is confused. His heart rate is 90/min, RR 18/min, SpO2 96% on O2 2 l/min via nasal prongs. 55 year old, admitted for an elective cholecystectomy tomorrow. He has a past history of epilepsy and had a generalized convulsion lasting for 1 minute which stopped spontaneously. The nurses report that his airway is patent, he is breathing, they have already turned him to the recovery position and his GCS is now 7.
B,A,D,C
34
A 23 year old, 60 kg asthmatic presents with increasing shortness of breath and wheeze over 1 day. On examination she is alert and able to complete full sentences, is not using her accessory muscles of respiration and has a respiratory rate of 22 breaths per min. Her heart rate is 98/min, pulsus paradoxus 5 mmHg, PEFR 220 l/min. She has received no treatment apart from 2 puffs of a salbutamol inhaler 4 hourly. Appropriate management would be: Select one: a. treat with intravenous bronchodilators and intravenous steroids and admit to medical ward for observation b. treat with inhaled bronchodilators and intravenous steroids and refer to ICU for ventilation c. treat with inhaled bronchodilators and oral steroids and discharge home d. treat with inhaled bronchodilators and intravenous steroids and admit to medical ward for observation e. treat with inhaled bronchodilators and discharge home
D
35
You were called to assess a patient who has become drowsy on day 1 post-laparotomy who had been receiving morphine for pain control. His SpO2 is 88% on room air breathing with a respiratory rate of 12 breaths per minute. His blood pressure is 120/80, pulse regular at a rate of 100 beats/minute. The patient makes incoherent sounds and withdraws his limbs to painful stimulation. What should be the next step of your management? Select one: a. Insert an oropharyngeal airway + 10L/minute oxygen via Hudson mask b. Head-tilt chin-lift + 10L/minute oxygen via Hudson mask c. 15L/minute oxygen via reservoir face mask d. Place the patient in recovery position e. Intubate the patient and give 100% oxygen
B
36
A hypotensive patient has warm peripheries, heart rate of 120/min and JVP 0. The most likely cause of shock is: Select one: A. Sepsis B. Myocardial infarction C. Hypovolaemia D. Ventricular tachycardia E. Massive pulmonary embolus
A
37
A 34 year old man with asthma and pneumonia has progressive shortness of breath associated with fever and productive cough for 2 days. He has been receiving Augmentin. His clinical presentation is: Blood pressure 110/50mmHg, pulse 110 beats/minute. Respiratory rate is 20 breaths/minute. SpO2 is 82 % on 4 L/min oxygen (Hudson mask). On examination of the lungs, there is bilateral expiratory wheeze and left lower zone bronchial breath sounds with percussion dullness. ABG shows pH 7.34, PCO2 3.8 kPa (29 mmHg), PO2 7.0 kPa (53 mmHg), HCO3 18 BE -4. High doses of salbutamol and ipratropium inhalers have already been started. What should be the immediate next step of your management? (P.136) Select one: a. Consult ICU b. Give 15L/min O2 via reservoir facemask c. Give hydrocortisone intravenously (Correcting hypoxia should be your immediate priority, the hydrocortisone can come (slightly) later) d. Save cultures and give iv antibiotics e. Perform a chest X-ray
B
38
You see a patient collapse on the ward. What will be your next action? (P. 68) Select one: a. Check his blood pressure b. Check his pulse (wrong) c. Phone for cardiac arrest team. d. Shout for help
D
39
You are called urgently to assess a patient who deteriorated after an ultrasound guided drainage procedure. On arrival, you find this patient is drowsy with GCS 10/15 and SBP 50mmHg. What is your next step of management? Select one: a. Give adrenaline 1mg iv b. Give adrenaline 0.1mg iv c. Give pack cell 1 unit full rate d. Give normal saline 500ml full rate e. Give gelofusion 500ml full rate
B
40
You have been asked to see four patients. What order would you see the patients in? A fully conscious 56 year old man with BP 70/40, HR 130/min, RR 30/min, patent airway. You were previously phoned about this patient about 5 minutes ago when his BP was 95/60. A 66 year old 70 kg man who underwent a radical gastrectomy this afternoon who has has been oliguric for an hour (30 ml in past hour). Airway patent, RR 21/min, HR 90/min, BP 140/80, SpO2 98% on 3 l/min oxygen via nasal prongs A 23 year old patient with asthma. Her heart rate is 95/min, respiratory rate 28/min, PEFR 180 L/min, BP 150/100, fully conscious A 54 year old woman who has had a generalized convulsion. Her GCS is V2, E1, M2. Her airway is patent, RR 10 breaths per minute, SpO2 95%, BP 160/110, HR 120/min.
A,C,D,B
41
You are asked to review a 70 year old patient who is 2 days post-hemicolectomy because he is febrile with a temperature of 39 C. He tells you he feels okay. On examination he looks well, heart rate 105/min, BP 140/70, heart sounds normal, warm peripheries, RR 15/min, SpO2 96% on room air, decreased breath sounds at both bases, abdomen soft, urine output 260 ml over past 4 hours. His laparotomy wound looks clean. His blood results from that morning show white blood cell count of 15. Your next steps should be: Select one: a. Take cultures b. Take cultures and obtain a chest X-ray, then review c. Take cultures and then start broad spectrum antibiotics d. Take cultures, start broad spectrum antibiotics and then obtain a chest X-ray e. Reassure the patient that fever due to atelectasis is common and there is no need for any further action
B
42
A 55 years old obese man is admitted to the neurosurgical ward 4 hours ago because of head injury after fall from height. His GCS drop from 14/15 to 5/15. His airway is patent. His respiratory rate is 6/min now. His oxygen saturation is 85% with 4L/min O2 through nasal cannula. What is your next step of management? Select one: a. Proceed to direct laryngoscopy intubation. b. Increase Oxygen to 15L/min with non-rebreathing mask c. Increase Oxygen to 6L/min with simple oxygen mask d. Do a blood gas and order a CXR e. Perform bag valve mask ventilation.
E (respiratory depression case, 6/min)
43
65 year old man, with a history of epilepsy, now has decreased consciousness following a generalized convulsion while lying in bed. His convulsion has stopped spontaneously but he is making loud snoring noises and has a see-saw breathing pattern. His SpO2 is 98% on 6l/min oxygen. Your immediate management should be: Select one: a. Insert oropharyngeal airway b. Triple manoeuvre c. Insert nasopharyngeal airway d. Intubate the patient e. Head-tilt chin lift
E (See-saw breathing pattern: ineffective breathing effort due to muscle fatigue or neuromuscular impairment)
44
A 45 year old woman (60kg) was admitted to the medical ward for liver abscess yesterday. She has had a urine output of 40 ml in the past 4 hours and she is agitated now. The following is her passive leg raising result, (92/43 -> 103/54) What is your next step(s) of management? Select one or more: [need one more answer] a. Give iv adrenaline 0.1mg b. Normal saline 100ml as fast as possible c. Start dopamine infusion d. Normal saline 1L as fast as possible e. Consult ICU for assessment
C,E Explanation: Mx of sepsis: IV antibiotics, blood culture Resuscitation: not fluid responsive ICU more expertise in dopamine titration
45
A 64 year old, 70kg diabetic man presents with repeated vomiting and diarrhoea, has a blood pressure of 82/43mmHg and a pulse rate of 113 /min on arrival to the medical ward. After passive leg raising, his BP is 105/55. He is mildly confused and his peripheries are cold. He has been given 2L Hartman’s solution intravenously in the emergency department. His JVP is hardly visible. What will be your next step of management? Select one: a. Start dopamine infusion according to body weight and infusion chart b. Give 0.1mg iv adrenaline c. Give 1L normal saline d. Start noradrenaline infusion according to body weight and infusion chart e. Give 500ml gelatin solution
C Still hypovolemic shock -> continue to correct by fluid therapy
46
The best strategy to avoid mistakes is maintain high vigilance at all times Select one: True False
False
47
A 50 year old woman with chest pain and ST elevation in V1-6, I, II develops shock with BP 70/40, CVP 10, HR 120/min, cold peripheries. Her chest is clear on auscultation. The appropriate supportive management is: Select one: A. Epinephrine 1mg IV bolus B. Fluid challenge of 250 ml of crystalloid C. Fluid challenge of 100 ml of crystalloid D. Dobutamine infusion E. Fluid challenge of 1000 ml of crystalloid
C
48
A patient who presents with central crushing chest pain, cold peripheries, hypotension, and decreased urine output is in your ward. You have ordered a dopamine infusion and the nurse has started the infusion at 5mL/h. (5 micrograms/kg/min). You should initially adjust the infusion rate to improve which of the following targets? Choose the best one. Select one: a. Oxygen saturation b. Heart rate c. Adequate diastolic blood pressure d. Adequate systolic blood pressure e. Urine output
C
49
While flying to your holiday destination there is a request for medical assistance. The cabin crew take you to see one of your fellow passengers is pulseless but is gasping. Your next step should be: Select one: a. Ask the cabin crew to bring an automatic external defibrillator b. The patient has gasping breathing efforts and so has not arrested and can be observed in the recovery position c. Start bag mask ventilation d. Immediately start mouth-to-mouth rescue breathing e. Lie the patient flat and start chest compressions
A
50
You have several patients under your care that require your attention. Match each patient with the order in which you would deal with him/her. [similar questions] Patient A: 20 year old man admitted with stridor due to acute epiglottitis. His initial respiratory rate was 30/min but has now fallen to 18/min and his stridor has disappeared. The nurse tells you that he appears to be sleeping Patient B: 52 year old man admitted by your MO with severe sepsis due to pneumonia. He has been resuscitated and now has the following observations: confused, urine output 15 ml over the past hour, pulse 130/min, BP 100/50, pH 7.28, PaCO2 2.8 kPa (21 mmHg), PaO2 13 kPa (98 mm Hg) on FiO2 0.5. The nurse rings you to tell you his blood gas results Patient C: 34 year old woman admitted with an acute asthmatic attack 5 hours ago. She is now deteriorating. Current observations: peak flow 150 l/min, respiratory rate 28/min, heart rate 105/min, BP 130/80, pulsus paradoxus 5 mm Hg, PaCO2 5 kPa (26 mm Hg), PaO2 12 kPa (165 mm Hg) on FiO2 0.4. You have been asked to reassess her Patient D: 45 year old man admitted with drowsiness and suspected community acquired meningitis. He has had a CT scan which shows no evidence of raised intracranial pressure and he has been started on high dose ceftriaxone. He is waiting for you to perform a lumbar puncture.
A,C,B,D
51
You have several patients under your care that require your attention. Match each patient with the order in which you would deal with him/her. [similar questions] Patient A: 20 year old man admitted with stridor due to acute epiglottitis. His initial respiratory rate was 30/min but has now fallen to 25/min and his stridor has disappeared. The nurse tells you that he is sleeping. Patient B: 52 year old man admitted by your MO with severe sepsis due to pneumonia. He has been resuscitated and now has the following observations: confused, urine output 35 ml over the past hour, pulse 130/min, BP 100/50, pH 7.28, PaCO2 2.8 kPa (21 mmHg), PaO2 13 kPa (98 mm Hg) on FiO2 0.5. The nurse rings you to tell you his blood gas results Patient C: 34 year old woman admitted with an acute asthmatic attack 5 hours ago. She is now deteriorating. Current observations: peak flow 50 l/min, respiratory rate 38/min, heart rate 115/min, BP 150/80, pulsus paradoxus 15 mm Hg, PaCO2 8 kPa (60 mm Hg), PaO2 12 kPa (165 mm Hg) on FiO2 0.4. You have been asked to reassess her Patient D: 45 year old man admitted with drowsiness and suspected community acquired meningitis. He has had a CT scan which shows no evidence of raised intracranial pressure and he has been started on high dose ceftriaxone. He is waiting for you to perform a lumbar puncture.
C,B,D,A
52
A 65 year old man with congestive heart failure was admitted for dyspnoea. On examination he was on 6L/min of facemask oxygen with SpO2 95% and BP 160/90. He had pitting oedema up to his knees and JVP at 4cm above clavicle. Chest examination showed bilateral crackles with mild use of accessory muscles. His urine output was 30mL/hour. How will you treat this patient? Select one: a. dopamine infusion b. fluid restriction only c. increase oxygen flow rate to 8L/min d. frusemide e. refer to renal team for dialysis
D
53
A 68 year old man presents with fever, productive cough and shortness of breath. He is a smoker but has no significant past medical history. He has a heart rate of 120/min, BP 180/100, respiratory rate of 6/min, is gasping, sweaty and drowsy. SpO2 82%% on 6L/min oxygen via simple facemask. The most appropriate immediate management is to: [similar questions] Select one or more: A. Give oxygen 15 l/min via reservoir facemask B. Call an anaesthetist to intubate and ventilate the patient C. Give oxygen 15 l/min via a self-inflating resuscitator D. Give oxygen 2 l/min E. Check his arterial blood gases
B,C
54
A 68 year old man presents with fever, productive cough and shortness of breath. He is a smoker but has no significant past medical history. He has a heart rate of 120/min, respiratory rate of 35/min, is using his accessory muscles and is sweaty. The most appropriate immediate management is to: [similar questions] (P. 18) Select one: A. Give oxygen 15 l/min via a reservoir facemask B. Give oxygen 2 l/min via a simple oxygen facemask C. Check his arterial blood gases D. Call an anaesthetist to intubate and ventilate the patient
A
55
While you are eating in a restaurant a fellow diner starts to choke and you are asked to help. He is conscious, grasping his throat and making coughing efforts but no sound can be heard. The most appropriate course of action is to: (P. 30) Select one: A. Attempt mouth-mouth ventilation B. Open his mouth and try to remove the foreign object C. Apply an abdominal thrust D. Encourage him to continue coughing efforts E. Apply a back blow
E
56
A 68 year old man with history of bronchiectasis and nephrotic syndrome was admitted with fever, productive cough, shortness of breath for 3 days. His temperature is 39.4°C, his respiratory rate is 25/minute, SaO2 96% on 50% O2 mask, CVP10, BP 90/40 before and 100/40 after passive leg raising. He has chronic peripheral edema. The chest X-ray is pending. What is the most appropriate resuscitation? Select one: [similar questions] a. 500 ml hetastarch as fast as possible b. Epinephrine 0.1 mg IV bolus c. 1000 ml 5% glucose as fast as possible d. Norepinephrine infusion e. 1000 ml Hartmann’s solution as fast as possible
E
57
A 68 year old man with history of bronchiectasis, poorly controlled hypertension and nephrotic syndrome was admitted with fever, productive cough, shortness of breath for 3 days. His temperature is 39.4°C, his respiratory rate is 25/minute, SaO2 94% on 50% O mask, CVP10, BP 95/40 before and 100/40 after passive leg raising. He has chronic peripheral edema. The chest X-ray is pending. What is the most appropriate resuscitation? Select one: [similar questions] (P. 56) a. Norepinephrine infusion b. 1000 ml 5% glucose as fast as possible c. 1000 ml Hartmann’s solution as fast as possible d. 500 ml hetastarch as fast as possible e. Epinephrine 0.1 mg IV bolus
A
58
A 68 year old man with history of bronchiectasis and nephrotic syndrome was admitted with fever, productive cough and shortness of breath for 3 days. His temperature is 39.4°C, his respiratory rate is 35/minute, SaO2 96% on 50% O mask, CVP10, BP 90/40 before and 95/50 after passive leg raising. He has chronic peripheral edema. The chest X-ray is pending. What is the most appropriate immediate action? Select one: [similar questions] a. 1000 ml Hartmann’s solution as fast as possible b. 1000 ml 5% glucose as fast as possible c. 500 ml hetastarch as fast as possible d. Epinephrine 0.1 mg IV bolus e. Dopamine infusion starting at 5 mcg/kg/min
E
59
A 68-year-old man fell in the toilet of the medical ward. You are informed by the nurse to assess the patient. He is lying motionless with no eye opening or verbal response. He flexes his limbs when stimulated. His breathing rate is 6 breaths per minute. SpO2 is 85% on room air. What is (are) your next step(s) of management? Select one or more: a. Place the patient in the recovery position b. Use simple face mask with oxygen flow of 6L/min c. Call for help d. Use reservoir face mask with oxygen flow of 15L/min. e. Use bag valve resuscitator to ventilate him with oxygen flow of 15L/min
C,E
60
A 55 year old man presents with respiratory failure worsening over a few days. On arrival he has a SpO2 85% on room air but this improves to 93% on 15L/min oxygen via reservoir face mask. The likely cause of his respiratory failure is: Select one: a. Pulmonary embolus b. Pneumothorax c. Pneumonia d. Acute pulmonary oedema e. Chronic obstructive pulmonary disease
C His saturation is below normal despite high flow oxygen so his A-a gradient must be high. This is not compatible with uncomplicated COPD.
61
A 70 kg diabetic patient being treated for acute cholecystitis develops acute confusion. His BP 100/60 , JVP 8 cm above sternal angle, heart rate 100/min, cold mottled peripheries, pH 7.3, PaCO2 3.9(29 mmHg), PaO2 13.2 (99 mmHg), urine output 20 ml/h. He has not responded to 100 ml normal saline over 10 minutes. The most appropriate haemodynamic intervention is: Select one: A. Dobutamine infusion B. Dopamine infusion C. Hartmann's solution 500 ml over 30 mins D. Frusemide E. None required
Explanation: blood pressure is tolerable to give dobutamine, in septic shock we aim MAP>65mmHg, dobutamine is vasodilator, helps more in terms of tissue perfusion
62
A 50-year-old obese man has collapsed in front of you with cardiac arrest. Help is on its way and you have decided he now requires mouth-to-mouth ventilation. Which of the following statements is true? Select one: a. If initial attempts at ventilation are not successful, you should delay further compressions and persist until ventilation is successful, or help arrives. b. If on the first attempt the chest does not rise normally, you may then attempt the “jaw thrust manoeuvre” to open the airway c. Always open the airway with the “jaw thrust manoeuvre” first d. Blow into the patient’s mouth steadily over 5 seconds while watching the chest rise e. You can press on the chest during expiration to improve exhalation
B
63
Which of the following responses to passive leg raising suggest that the patient is likely to be fluid responsive? The heart rate is 100/min and BP is 100/50 (MAP 74) before passive leg raising Select one: a. MAP 85 after passive leg raising b. BP 105/45 after passive leg raising c. Heart rate 90/min after passive leg raising d. BP 100/80 after passive leg raising e. BP 115/70 after passive leg raising
B
64
The most effective way to prevent harm to patients is to eliminate mistakes Select one: True False
False
65
A 50 year old man presented with perforated peptic ulcer. The following are all signs of sepsis except: (P. 54) Select one: a. oliguria b. desaturation c. Hb 7 d. hypotension e. confusion
C (Whilst many patients develop low Hb after fluid resuscitation, it is not a feature of sepsis.)
66
You have been informed about four patients. Which order would you see them in? Patient A 54 year old woman admitted from the emergency department with breathlessness. Her arterial blood gas on room air shows pH 7.1, PaCO2 1.5 kPa (11 mmHg), PaO2 9 kPa (68 mmHg). The nurse has already started giving oxygen 6 l/min via facemask. Patient B 72 year old man with 10 minutes of central crushing chest pain which resolved with sublingual GTN. Patient C 84 year old woman with pneumonia. Her respiratory rate is 30/min, SpO2 94% on 15L/min oxygen via reservoir facemask, BP 110/40, urine output 20 ml over past hour. The nurse asks you to see her because she is confused and agitated. Patient D 65 year old man with a history of an abdominal aortic aneurysm who has BP 60/40 and heart rate 130/min
D,A,C,B Patient A has very severe metabolic acidosis and is already compensating maximally Patient B sounds relatively stable Patient C is shocked but not hypotensive Patient D is about to die
67
A 70 year old woman with acute pulmonary edema has a respiratory rate of 30/min, heart rate 120/min, BP 170/90 and SpO2 91% despite oxygen 15L/min via a reservoir facemask, frusemide IV and IV glyceryl tri-nitrate infusion. On auscultation of her chest she has bilateral crackles. What would you do? Select one or more: a. Start continuous positive airway pressure non-invasive ventilation b. IV morphine c. Start bilevel airway pressure non-invasive ventilation d. Intubate and ventilate e. Increase dose of glyceryl trinitrate
A,B,E Explanation: The patient has cardiogenic pulmonary oedema with moderately severe hypoxia despite high flow oxygen and hypertension. You need to increase her supportive therapy and maximize the treatment of her heart failure. CPAP should be tried before BiPAP for cardiogenic pulmonary oedema. Her hypertension is probably due to high sympathetic drive due to the discomfort of pulmonary oedema. It will worsen her heart failure due to high afterload. IV morphine will reduce the discomfort. GTN will reduce her preload and, to a lesser extent, the afterload. The major contraindication is hypotension, but she is not hypotensive. It is very short acting so its effect will disappear rapidly if the patient's BP falls after giving morphine.
68
You are called to see a 65 year old woman immediately after admission from A&E. She tells you that she became progressively short of breath over the past few days. Today, after finishing her lunch she felt too breathless to continue and came to hospital. Her BP is 125/90 mmHg and pulse is 110/min. She has respiratory rate 26/min, decreased breath sounds in the left middle and lower zones and dullness to percussion in the same area. She is receiving oxygen via a reservoir face mask (15 l/min). Her arterial blood gas shows a pH = 7.39 PaO2 = 9 kPa, PaCO2 = 4.5 kPa. The most likely diagnosis is: Select one: [version 1] a. Pleural effusion b. Foreign body upper airway obstruction c. Pulmonary embolus d. Severe COPD exacerbation e. Aspiration pneumonia
A (Wrong, this is possible but there is nothing in the history to suggest she is at increased risk of aspiration and the signs are not classical for pneumonia)
69
You are called to see a 65 year old woman immediately after admission from A&E. She tells you that she became suddenly short of breath earlier during the afternoon while waiting for a subway train, on her way home after going out to lunch. She is receiving oxygen via a Hudson mask (10l/min). Her BP is 95/60 mmHg and pulse is 130/min. Her arterial blood gas shows a pH = 7.32, PaO2 = 9 kPa 68 mm(68 mmHg), PaCO2 = 3.5 kPa (26 mmHg). The most likely diagnosis is: Select one: [version 2] a. Pleural effusion b. Aspiration pneumonia c. Severe COPD exacerbation d. Foreign body upper airway obstruction e. Pulmonary embolus
E A: (wrong, the speed of onset is very fast (subway trains come every few minutes), which makes pleural effusion very unlikely) E: (She clearly has a high A-a gradient (her PaO2 is only 9 kPa [68 mmHg] despite high flow oxygen) and the speed of onset is very fast (subway trains come every few minutes). In addition she is hypotensive)
70
You are called to see a 65 year old woman immediately after admission from A&E. She became progressively short of breath earlier during the afternoon while shopping. She is receiving oxygen via a Hudson mask (10l/min). Her BP is 120/90 mmHg and pulse is 110/min. Her arterial blood gas shows a pH = 7.32, PaO2 = 9 kPa, PaCO2 = 6.5 kPa, bicarbonate 18 mmol/l. The most likely diagnosis is: Select one: [version 3] a. Pleural effusion b. Pneumothorax c. Acute severe asthma d. Severe COPD exacerbation e. Acute heart failure
E A: (wrong, the speed of onset is not typical for a pleural effusion) D: (wrong, because the speed of onset is not typical for an exacerbation of COPD and the PaO2 is only 9 kPa (68 mmHg) despite high flow oxygen, suggesting a high A-a gradient) E: The PaO2 is only 9 kPa (68 mmHg) despite high flow oxygen, suggesting a high A-a gradient and the patient has a mixed metabolic and respiratory acidosis so the blood gas is classical for a patient with severe acute heart failure. In addition the speed of onset is compatible.
71
You have witnessed a man suddenly collapse. He is pulseless and but is gasping. There is no one nearby. The correct management is: Select one: a. Call for help, but as there is nobody near, you should pause to use you mobile phone to phone for help from an emergency services, and then start chest compressions b. Call for help, but if no one responds, do not leave the patient to get help and immediately start chest compressions c. Leave the patient to obtain an automatic external defibrillator as early defibrillation is the key to survival d. The patient has gasping breathing efforts and so has not arrested and can be observed in the recovery position e. Immediately start mouth-to-mouth rescue breathing
A
72
You have been asked to see 4 patients. Which order would you see them in? 55 year old, admitted for an elective cholecystectomy tomorrow. He has a past history of epilepsy and had a generalized convulsion lasting for 1 minute which stopped spontaneously. The nurses report that his airway is patent, he is breathing, they have already turned him to the recovery position and his GCS is now 7. 65 year old, 60 kg patient who underwent an elective total colectomy 4 days ago. You have been called because he is confused. His heart rate is 90/min, RR 18/min, SpO2 96% on O2 2 l/min via nasal prongs. 32 year old asthmatic who was admitted to a medical ward 10 hours ago. She has been treated with a salbutamol inhaler 4 puffs hourly, ipratropium inhaler 4 puffs 6 hourly and IV hydrocortisone 200 mg 8 hourly. Her peak flow has fallen from 120 l/min on admission to 80 l/min in the past 4 hours. Her PaCO2 is 5 kPa (38 mmHg). Her respiratory rate is 30/min, heart rate 120/min, SpO2 100% on 4 l/min oxygen via a simple oxygen mask. 62 year old man admitted with chest pain and ST elevation in the anterior ECG leads. He has already been started on aspirin and has received thrombolysis. The nurse rings you to tell you that his heart rate is 125/min, BP 70/40, SpO2 92% on 15 l/min oxygen through a non-rebreathing mask, respiratory rate 10/min and he is drowsy.
D,C,A,B
73
A patient with septic shock has the following results. BP 110/40 (mean 60), CVP 8 mmHg before fluid challenge and 14 mmHg after, urine output 10 ml/h, pH 7.34, bicarbonate 18 mmol/l, K+ 5.0 mmol/l, creatinine 390 µmol/l. The most appropriate initial treatment is: [similar questions] Select one: A. Low dose dopamine infusion B. Frusemide C. Norepinephrine infusion D. Fluid bolus 300 ml colloid E. Renal replacement therapy
A: (Wrong, this does not prevent acute renal failure) C: (his MAP is too low) D: (wrong) E: (wrong)
74
A patient with septic shock has the following results. BP 110/40 (mean 60), CVP 8 mmHg, urine output 10 ml/h, pH 7.34, bicarbonate 18 mmol/l, K+ 5.0 mmol/l, creatinine 390 µmol/l. The most appropriate initial treatment is: [similar questions] Select one: A. Frusemide B. Norepinephrine infusion (wrong, Although the BP is a little low norepinephrine is not the first choice for either hypotension or oliguria in this situation) C. Low dose dopamine infusion D. Renal replacement therapy E. Fluid bolus 300 ml colloid
E
75
A patient with septic shock has the following results. BP 110/40 (mean 60) before passive leg raising and 115/45 after, urine output 10 ml/h, pH 7.34, bicarbonate 18 mmol/l, K+ 5.0 mmol/l, creatinine 390 µmol/l. The most appropriate initial treatment is: Select one: A. Frusemide B. 50 ml glucose 50% and 10 U insulin IV bolus C. Norepinephrine infusion D. Renal replacement therapy E. Fluid bolus 300 ml colloid
C A: (wrong, “Is his problem fluid overload?”, Ans should be NE infusion) Is his immediate problem fluid overload?
76
A 70 year old man with a history of ischaemic heart disease is admitted to your ward following endoscopic banding for bleeding oesophageal varices. Three days later he becomes hypotensive. His heart rate 130/min and his jugular venous pressure is 0 cm above the sternal angle. He has cold peripheries. The most likely cause of his shock is: Select one: A. Haemorrhagic shock B. Pulmonary embolus C. Distributive shock due to liver failure D. Cardiogenic shock E. Septic shock
A
77
A 60 year old man was admitted with vomiting and diarrhoea. His blood pressure is 80/60 and he is awake but drowsy. His JVP is not visible and capillary refill time is 2 seconds. After admission he was still vomiting. How will you resuscitate this patient? Select one: a. IV 0.9% Saline 100mL over 15 minutes b. IV 0.9% saline 250 ml as fast as possible c. IV 0.9% Saline 40mL/hour d. IV 0.9% Saline 1 L as fast as possible e. regular oral rehydration fluid
D
78
A previously well 48 year old patient presents with cellulitis of the left foot. She is febrile and has a raised white cell count. She was given intravenous Augmentin (amoxicillin/clavulanate) two hours ago. Her BP is 86/24, pulse 126/min and urine output 100ml over the last 4 hours. Her hands are warm to the touch. Her JVP is just visible sitting up. You have noticed a new petechial rash over her chest and abdomen. The correct management is: (P. 95) Select one: a. Start a rapid infusion of 50-100ml normal saline, and consider adding a Dopamine infusion starting at 5μg/kg/min b. Immediately stop the antibiotic, administer IM Adrenaline 0.4mg, and call the ICU for help c. Start a rapid infusion of 500-1000ml normal saline, and consider adding a Dobutamine infusion starting at 5μg/kg/min d. Immediately stop the antibiotic, administer IV Adrenaline 0.1mg, and call the ICU for help e. Start a rapid infusion of 500-1000ml normal saline, and consider adding a Dopamine infusion starting at 5μg/kg/min Explanation: petechial rash points towards sepsis; if it is anaphylaxis, more likely would give maculopapular rash
E Explanation: petechial rash points towards sepsis; if it is anaphylaxis, more likely would give maculopapular rash
79
A confused patient is receiving oxygen 4 L/min via a simple facemask. Which of the following would suggest that the patient is NOT critically ill Select one: a. Blood pressure 110/60 b. Respiratory rate 15/min c. pH 7.3, PaCO2 2.1 kPa (15.75 mmHg), PaO2 10 kPa (75 mmHg) d. Pulse oximetry 97% e. Heart rate 115/min
B
80
You see a 55 year old man who has recently had a right pleural biopsy. He is sweaty, has cold peripheries, SpO2 not detectable, cyanosed, BP 90/50. Chest expansion is reduced on the right, breath sounds reduced on right, trachea deviated to left. Immediate management is: (P. 19) Select one: a. Needle thoracostomy left second intercostal space, mid clavicular line b. Intercostal drain, right fifth intercostal space, just anterior to mid axillary line c. Needle thoracostomy right second intercostal space, parasternal d. Needle thoracostomy right second intercostal space, mid clavicular line e. Needle thoracostomy right second intercostal space, anterior axillary line
D
81
A 31 year old male patient has been admitted to the emergency room. He has a history of seizures and is receiving regular medication. He is now lying on his back, deeply unconscious. The nurse has placed a pulse oximeter probe to the patient that shows an arterial saturation of 92%. His pulse is 110 beats per minute and BP is 140/90 mmHg. Further examination reveals that his chest is making vigorous breathing movements. You can feel some air coming from the mouth during expiration. The nurse has already placed a Hudson type oxygen mask on his face (about 5 minutes ago). The best management response would be to: Select one: a. Insert an oropharyngeal airway. b. Immediately apply the triple manoeuvre (jaw thrust) and insert an oropharyngeal airway. c. Gently change to 100% oxygen via a non-rebreathing mask. d. Immediately apply the head-tilt, chin lift manoeuvre and place the patient in the recovery position. e. Immediately apply the triple manoeuvre (jaw thrust) and place the patient in the recovery position
D
82
A 42 year old male patient has been admitted to the emergency room. There is no history available. He is lying on his back, deeply unconscious. A pulse oximeter is attached to the patient and shows an arterial saturation of 78%. His pulse is 120 beats per minute and BP is 100/70 mmHg. Further examination reveals that his chest is making vigorous breathing movements. There is no sound coming from the mouth or airway. The nurse has already placed a Hudson type oxygen mask on his face (about 3 minutes ago). What should you do first? Select one: A. Place a nasogastric tube to empty the stomach B. Triple airway manoeuvre (wrong, This is used when other methods have failed to open the airway) C. Immediately place the patient in the recovery position D. Place an oropharyngeal airway E. Apply the head-tilt, chin lift manoeuvre
E
83
You are called to see a patient who was recently discharged from the ICU. He is complaining of shortness of breath. While in the ICU a single lumen cuffed tracheostomy tube was inserted because the patient required prolonged intubation. On examination he is using accessory muscles, has intercostal and subcostal recession and has a see-saw pattern of breathing. Pulse 130/min, BP 200/120, respiratory rate 35/min, SpO2 86% on O2 6l/min via tracheostomy mask. The most appropriate immediate action is: (P. 31) Select one: a. Call for help and give 100% oxygen via a bag valve resuscitator b. Deflate the tracheostomy cuff c. Attempt to pass a suction catheter down the tracheostomy tube d. Remove the tracheostomy tube and give oxygen via a facemask e. Arrange non-invasive ventilation
A
84
A 23 year old man with a history of epilepsy and alcoholism was admitted for general tonic clonic seizure witnessed at home 20 minutes ago. When you assess him now in the emergency department, he remains drowsy. There is no limb twitching or uprolling eyes. His SpO2 is 85% on 6 L/min of oxygen. His airway is patent and his respiratory rate is 20 breaths/minute. His blood pressure is 100/60mmHg, his heart rate is 95 beats/minute. What should be the next step of your management? (P.44?) Select one: a. Give thiamine and dextrose intravenously b. Give 15 l/min oxygen via reservoir mask / Give 100% oxygen via facemask c. Give a loading dose of phenytoin intravenously d. Place the patient in the recovery position e. Arrange an urgent CT brain
B
85
You have been asked to see 4 patients. Give the order in which you would see them. 53 year old who has just presented with haematemesis. HR 120/min, BP 90/60, SpO2 97% on room air, fully conscious 34 year old asthmatic who presented with acute shortness of breath a few hours ago. The nurses report that she is now drowsy, respiratory rate 8/min, BP 90/40, heart rate 60/min 23 year old, previously well, woman who presented to the emergency department with glucose 32 mmol/l, pH 7.2, ketones +++, SpO2 99% on room air, RR 28/min, haemodynamically stable and fully conscious 67 year old woman with a serum sodium 107 mmol/l. She is a little confused but is otherwise well.
B,A,C,D
86
A 55 year old man patient was admitted for urinary tract infection with fever, heart rate of 110 beats/minute and blood pressure 85/45 mmHg, his urine output is only ~10ml in the one hour since admission. His potassium is 5 mmol/l, urea 15 mmol/l and creatinine 280 mcmol/l, pH 7.35 BE -4 . What would you do next in addition to fluid resuscitation? Select one: a. Perform culture and give antibiotics b. Arrange for renal replacement therapy c. Arrange for urgent imaging of urinary system d. Give bicarbonate e. Give furosemide
A
87
A 30 year old woman with a history of anxiety presents with sudden onset of dyspnoea. Her respiratory rate is 24 and the rest of the examination is normal. An ABG on room air showd: pH 7.44, PO2 9 kPa (67.5 mmHg), PCO2 3.5 kPa (26.25 mmHg), HCO3 23 mmol/L, BE -1. The likely diagnosis is: Select one: a. venous blood gas b. anxiety/hyperventilation c. pulmonary embolism d. benzodiazepine withdrawal e. diabetic ketoacidosis
C Explanation: There is an A-a gradient of ~6.6 kPa (~49 mmHg) which is abnormal. Thus this cannot be attributed to anxiety alone.
88
You are asked to see a patient who is being treated for meningitis because she has become short of breath. She eye opens to speech, is confused but her voice is normal when speaking. Her respiratory rate is 30/min, she is using accessory muscles and is sweaty, tachycardic and hypertensive. She has crackles in the left base and her saturation is 90% on 6 l/min oxygen. Your immediate management should be: Select one: (P. 18) a. Perform a head tilt, chin lift b. Insert nasopharyngeal airway c. Bag mask ventilate with 100% oxygen d. Give 15 l/min oxygen via reservoir facemask e. Insert oropharyngeal airway
D (severe desaturation as evidenced by presence of sympathetic response, changing mask can be done in short time) Explanation: airway treatment not needed because voice is normal, and patient can speak
89
You are resuscitating a patient from a cardiac arrest. You have delivered a shock and resumed chest compressions. Useful monitoring and investigations at this point include: Select one or more: a. Pulse oximetry (pointless during chest compression) b. Non invasive blood pressure (pointless during chest compression) c. Electrolytes d. Arterial blood gas e. 12 lead ECG (pointless during chest compression)
C,D
90
A 75-year-old man was admitted to the medical ward for urinary tract infection 3 days ago. He is found to have a fall from his bed by the nurse. His GCS is 5/15 and you are called to assist in managing this patient. After given simple mask with 6L/min Oxygen to him, his oxygen saturation is 85% and his breathing rate is 2 breaths per minute with mist seen on the mask in every breath. What is your next step of management? Select one: a. Increase Oxygen to 15L/min with non-rebreathing mask b. Perform bag valve mask ventilation. c. Proceed to direct laryngoscopy intubation d. Increase Oxygen to 8L/min with simple oxygen mask e. Do a blood gas and order a CXR
B A: (wrong, It is not a problem or oxygenation, but ventilation.) B: (Bag valve mask ventilation can be provided by intern easily to support this patient's ventilation.)
91
A 70 year old diabetic patient as admitted with fever, right upper quadrant abdominal pain and vomiting of undigested food. His blood pressure is 60/35mmHg, pulse 140/minute , temperature 39.6 °C, his peripheries are warm. He is jaundiced and he has tenderness over the right upper quadrant. After giving 2 litres of lactated Ringer’s solution, the blood pressure rises to 110/60, pulse 102 but he is drowsy and the serum lactate remains high. Which is the ideal vasopressor/inotrope? Select one: [similar questions] a. dobutamine b. norepinephrine c. levosimendin d. epinephrine e. dopamine
A
92
A 70 year old diabetic patient as admitted with fever, right upper quadrant abdominal pain and vomiting of undigested food. His blood pressure is 60/35mmHg, pulse 140/minute , temperature 39.6 °C, his peripheries are warm. He is jaundiced and he has tenderness over the right upper quadrant. Despite giving 1 litre of lactated Ringer’s solution, the blood pressure remains 80/40, pulse 132 and he is drowsy. Which is the ideal vasopressor/inotrope? Select one: [similar questions] a. dobutamine b. epinephrine c. levosimendin d. dopamine e. norepinephrine
E
93
An adequate preload is indicated by: Select one or more: a. Rise in CVP of 4 mmHg after a fluid challenge b. CVP 14 mmHg (wrong, do not know the patient's ventricular compliance) c. Change in BP from 90/60 to 100/70 on passive leg raising d. Change in BP from 90/60 to 95/60 on passive leg raising (wrong) e. Rise in CVP of 1 mmHg after a fluid challenge
A,C
94
A 65 year old woman presents with a myocardial infarction. She is cyanotic with cold clammy skin and is confused. Her heart rate is 100/min, blood pressure 110/90. She has not responded to an infusion of 50 ml of Normal saline over 5 mins. The most appropriate immediate treatment is: Select one: A. Noradrenaline infusion 5 mcg/min B. Epinephrine 0.1 mg IV bolus C. Dopamine infusion 5 mcg/kg/min D. Dobutamine infusion 5 mcg/kg/min E. Normal saline 500 ml over 15 mins
D
95
The correct technique of chest compressions is (choose all correct answers): Select one or more: a. Never allow full chest recoil between compressions b. The chest compression rate should be 60/min c. Compression-ventilation ratio should be 30:2 until the patient is intubated d. Compression-ventilation ratio should be 30:2 until an oral airway is inserted e. The depth of compression should be 10mm f. The chest compression rate should be 100/min
C,F
96
You find a patient collapsed in a busy shopping mall. He is unconscious and pulseless but is taking gasping breaths. There are no safety issues. You should immediately: Select one or more: a. Shout for help b. Perform a triple manoeuvre c. Start chest compressions d. Turn him to the recovery position (wrong, he has had a cardiac arrest. It is not possible to provide effective chest compressions in the recovery position) e. Leave him to find an AED
A,C
97
You have been asked to see three patients. Which order would you see them in? 72 year old man who presents with confusion. A nurse rings you with his initial blood results: creatinine 350 mcmol/l, Na 107 mmol/l, K 5.4 mmol/l. 72 year old man who presents with malaise. A nurse rings you with his initial blood results: creatinine 350 mcmol/l, Na 137 mmol/l, K 7.4 mmol/l. 72 year old man who presents with malaise and chronic renal failure. A nurse rings you with his initial blood results: creatinine 950 mcmol/l, Na 137 mmol/l, K 5.4 mmol/l.
B,A,C
98
You are asked to see a hypotensive patient. He is a diabetic, has a history of hyperlipidaemia and has been complaining of chest pain. Heart rate 110/min sinus tachycardia, BP 80/50, JVP 8 cm above sternal angle, bilateral basal crackles. He is able to talk to you. The appropriate next step in the management of his shock is: Select one: a. Dopamine infusion b. Hartmann's solution 100 ml as fast as possible c. Hartmann's solution 250 ml as fast as possible d. Hartmann's solution 1000 ml as fast as possible e. Epinephrine 0.1 mg IV
A
99
The CXR of a 65 year old male smoker with coronary artery disease who presented to the Accident & Emergency department with sudden onset of shortness of breath coming on over a few minutes is shown. His heart rate is 110/min. (P. 80) [no CXR was shown lol] What is treatment should be given, in addition to high flow oxygen: Select one: A. Intercostal drain B. amoxicillin-clavulanate C. cefotaxime D. glyceryl trinitrate and morphine E. beta blocker
D
100
A previously well 54 year old man with a liver abscess develops oliguria. His urine output has been 5-10 ml/h for the past 4 hours despite fluid resuscitation. He is receiving appropriate antibiotics. His BP is 130/60, heart rate 110/min, JVP 5 cm above sternal angle, SpO2 96% on 2 l/min of oxygen through nasal cannulae. His BP on passive leg raising is 145/75. His blood results are as follows: pH 7.3 PaCO2 3.1 kPa PaO2 11 kPa Na 142 mmol/l K 5.2 mmol/l Urea 20 mmol/l Creatinine 305 mcmol/l The most appropriate next treatment is: Select one: a. Urgent drainage of liver abscess b. 50 ml 50% glucose and 10 U insulin c. Urgent renal replacement therapy d. 20 mg frusemide IV (wrong) e. 500 ml bolus of crystalloid
A
101
A 60-year-old woman is admitted for high fever and right loin pain. She is receiving iv antibiotics and normal saline 80ml/hr. She just come back from radiology department after her percutaneous drainage of her right pyonephrosis. Her latest blood pressure is 80/40 and you have just finished the following assessment at the bedside. (leg raising: 80/40 -> 98/52) Select one: a. Start dobutamine infusion b. Normal saline 1L as fast as possible c. Give iv adrenaline 0.1mg d. Give iv adrenaline 1mg e. Increase normal saline to 120ml/hr
B
102
A 22 year old woman presented with acute asthma to the Accident and Emergency Department. Initial assessment showed she was using accessory muscles, had bilateral wheezing and her respiratory rate was 30. She was able to speak in short sentences and was able to use inhalers using a chamber and face mask. She was given salbutamol, ipratropium bromide and IV hydrocortisone. You review her again in 15 minutes and on chest examination her wheezing was gone and her respiratory rate is 8. She is unresponsive. What is your response? (P. 41) Select one: a. Send her to the ward as she has stabilized b. Call psychiatry team as you think this patient is malingering c. Call the stroke team as you worry she may have had a stroke d. She is improving since her respiratory rate has decreased and the wheezing is gone e. Bag mask ventilate and call for help (ICU or Senior) as this patient has deteriorated
E
103
While you are eating a restaurant a fellow diner starts to choke and you are asked to help. He is conscious, grasping his throat and making coughing efforts. The most appropriate course of action is to: Select one: A. Encourage him to continue coughing efforts B. Attempt mouth-mouth ventilation C. Open his mouth and try to remove the foreign object D. Apply a back blow E. Apply an abdominal thrust
A
104
You are the house officer called to manage a 28-year old female with sudden loss of consciousness. She has a history of epilepsy and had a recent reduction in dose of phenytoin due to stable disease. On assessment, she is unarousable but not twitching. Her blood pressure is 160/90 mmHg, pulse is 99/min, respiratory rate is 25/minutes, SpO2 is 94% on room air and her peripheries are cold. Her airway is patent. Her blood spot glucose is 6. What is the most appropriate next action? Select one: a. Give midazolam/diazepam iv b. Put the patient in recovery position c. Give 10 L/minute oxygen via Hudson mask d. Give phenytoin e. Give 1L normal saline
B
105
You have several patients under your care that require your attention. Match each patient with the order in which you would deal with him/her. [similar questions] Patient A: 53 year old man with acute renal failure. Creatinine 230 mcmol/l, urea 24 mmol/l, K 7.1 mmol/l, Na 145 mmol/l, BP 130/70, heart rate 98/min, warm peripheries, respiratory rate 25/min, conscious, urine output 40 ml/h. You have just been informed of his results and observations. Patient B: 34 year old woman with a history of poorly controlled epilepsy. Has just had a generalized tonic clonic convulsion which terminated spontaneously after 2 minutes. She now has a GCS of 5 and has been placed in the recovery position. Her airway is patent and she is breathing with a saturation of 94% on room air. You have been asked to assess her. Patient C: 38 year old man with blood glucose 15 mmol/l, K 4.5 mmol/l, Na 135 mmol/l, respiratory rate 18/min, heart rate 90 mmol/l, BP 140/70, temperature 36.8°C. You have just been informed of his results and observations Patient D: 67 year old woman just admitted to your ward with respiratory failure. She is pyrexial (38.9°C), urine output 10 ml/h for past two hours, respiratory rate 30/min, SpO2 93% on non-rebreathing mask with 15 l/min oxygen, BP 100/50, heart rate 110/min. You have been asked to complete her admission assessment
A,D,B,C
106
You have several patients under your care that require your attention. Match each patient with the order in which you would deal with him/her. [similar questions] Patient A: 53 year old man with acute renal failure. Creatinine 230 mcmol/l, urea 24 mmol/l, K 6.4 mmol/l, Na 145 mmol/l, BP 130/70, heart rate 98/min, warm peripheries, respiratory rate 25/min, conscious, urine output 40 ml/h. You have just been informed of his results and observations. Patient B: 34 year old woman who had a generalized tonic clonic convulsion 2 hours ago. The convulsion terminated spontaneously after 2 minutes. She was placed in the recovery position and her GCS has improved slowly to 10. Her airway is patent and she is breathing with a saturation of 94% on room air. You have been asked to assess her. Patient C: 38 year old man with blood glucose 15 mmol/l, K 4.5 mmol/l, Na 135 mmol/l, respiratory rate 18/min, heart rate 90 mmol/l, BP 140/70, temperature 36.8°C. You have just been informed of his results and observations Patient D: 67 year old woman just admitted to your ward with respiratory failure. She is pyrexial (38.9°C), urine output 10 ml/h for past two hours, respiratory rate 30/min, SpO2 93% on non-rebreathing mask with 15 l/min oxygen, BP 100/50, heart rate 110/min. You have been asked to complete her admission assessment
D,A,B,C
107
Which of the following statements regarding in hospital resuscitation of cardiac arrest is/are correct? [version 1] Select one or more: a. Vasopressin should be used in preference to epinephrine b. Amiodarone is recommended for unstable, shock-resistant VF/VT c. Early defibrillation is the goal in VT/VF arrests d. The initial energy for monophasic defibrillation depends on the manufacturer of the defibrillator e. Successful resuscitation requires team work
B,C,D,E
108
Which of the following statements regarding resuscitation from cardiac arrest is correct? [version 2] Select one or more: a. The initial energy for monophasic defibrillation is 200J b. Amiodarone is recommended for unstable, shock-resistant VF/VT c. Early defibrillation is the goal in VT/VF arrests d. Vasopressin should be used in preference to epinephrine e. Successful resuscitation requires team work
B,C,E
109
A 64 year old man presents with sudden onset of shortness of breath. On examination he is sweaty with cold peripheries. His JVP is 8 cm above the sternal angle, BP 190/100, heart rate 120/min, heart sounds normal. He is gasping and is drowsy. On auscultation of his chest he has bilateral crackles throughout both lung fields. His SpO2 is 85% on 15L/min oxygen via a reservoir face mask. Immediate treatment should include: (p.81) Select one or more: a. Captopril b. Frusemide IV c. CPAP d. Refer for invasive mechanical ventilation e. BiPA
B,D
110
111
You are asked to see a previously well 65 year old woman who had a Whipple's operation one day ago. About 30 minutes ago she had a left subclavian central line inserted for total parenteral nutrition. Since then she has become short of breath. When you see her she is able to talk, SpO2 88% on 15 l/min oxygen via reservoir face mask, respiratory rate 40/min, heart rate 130/min, BP 70/40, cold peripheries, JVP 8 cm above sternal angle. the BP after passive leg raising is 80/50. You think that the breath sounds are reduced on the left, percussion is more resonant on left and the trachea is slightly deviated to the right. The immediate management is: Select one: a. Insert a cannula in the 2nd intercostal space in the left midclavicular line b. IV epinephrine 0.1 mg c. 100 ml normal saline as fast as possible d. Insert a cannula in the 3rd intercostal space in the left midclavicular line e. 1L normal saline as fast as possible f. Open the airway
A
112
The correct technique of chest compressions is (choose all correct answers): Select one or more: a. Compression-ventilation ratio should be 30:2 until the patient is intubated b. The heel of your hand should be placed just to the left of the sternum c. The chest compression rate should be 100-120/min d. The depth of compression should be at least 6 cm
A,C
113
A 75-year-old man was admitted to the orthopaedic ward two hour ago after removal of infected implant of his right hip join. Broad spectrum antibiotics and 1L of crystalloid was given. The nurse informs you that his blood pressure is 80/40 and heart rate is 115/min. His haemodynamic parameter after passive leg raising is BP 115/55 HR 95/min. His haemoglobin level after the operation is 6.5g/dL Two units of cross matched blood is ready in the ward. What is the most appropriate next step of management? (p.141) Select one: a. Give one unit of pack cell full rate b. Give 1mg intravenous adrenaline c. Give 500ml of crystalloid full rate d. Give 500ml of colloid full rate. e. Give 0.1mg intravenous adrenaline
A Explanation: in critically ill patients, transfusion threshold of 7 g/dl
114
A 70 year old male patient presents with a 3 day history of progressive shortness of breath. He is not cyanosed (pulse oximeter oxygen saturation 88% on room air). He is a chronic smoker (40 pack year smoking history). Clinically he is hyperinflated and has expiratory wheeze. You have an arterial blood gas (ABG) available to help your decision. The following is the best indicator for the need for ICU assessment and possible invasive mechanical ventilation: Select one: a. PaCO2 is 8.5 kPa measured on ABG b. PaO2 is 8.6kPa c. Arterial oxygenation saturation is 88% d. Respiratory rate is 26/min e. pH is 7.10
E
115
A 65 year old male patient presents with a 3 day history of progressive shortness of breath. He is currently cyanosed (pulse oximeter oxygen saturation 82% on room air). He is a chronic smoker (40 pack year smoking history), and suffers from chromic bronchitis. Clinically he is hyperinflated and has expiratory wheeze. The best immediate management, in the order of priority, should be: Select one: a. treat hypoxia with an oxygen mask (deliver 50% oxygen), then start antibiotics, then encourage removal of secretions by coughing, then start DVT prophylaxis b. start bronchodilators, then encourage coughing to remove secretions, then start DVT prophylaxis, then start antibiotics, then do an arterial blood gas (ABG) to confirm hypoxia c. treat hypoxia with an oxygen mask (deliver 50% oxygen), then start bronchodilators, then start steroids, then do an ABG to assess degree of acidosis d. start bronchodilators, then start steroids, than start DVT prophylaxis, do an arterial blood gas to confirm hypoxia and then start oxygen by mask (deliver 50% oxygen), only if necessary
C
116
A 55-year-old man is admitted to the medical ward for suspected drug overdose. He has no verbal response or eye opening on painful stimulation. He flexes his limbs on pain stimulation. The following is his blood gas result while on simple mask with 6L/min oxygen, pH: 7.14 pCO2: 8.2kPa (62mmHg) pO2: 9.2kPa (69 mmHg) What is (are) your next step(s) of management? Select one or more: a. Change to Hudson's mask with 8L/min oxygen b. Start non-invasive ventilation c. Use bag-valve resuscitator to ventilate him with 15L/min oxygen d. Refer ICU for intubation e. Change to reservoir face mask with 15L/min O2
C,D
117
A patient with sepsis has BP 100/40 (rises to 120/50 on passive leg raising), heart rate 120/min and oxygen saturation 97% (on FiO2 0.3). He is acidotic, oliguric and confused. He is in positive fluid balance of 3L over the first 12h of his hospital admission. The most appropriate next step in his resuscitation is: (P. 49) Select one: a. Start norepinephrine infusion b. Start dopamine infusion c. No further resuscitation is required d. 500 ml Hartmann’s solution over 30 minutes e. Start dobutamine infusion
D
118
You are walking with a friend in a deserted street when he collapses. You shout for help but no one hears you. He is unconscious and is making no breathing efforts. The most appropriate course of action is to: (P. 65)
Call for ambulance
119
An 80 years old patient with a history of ischaemic heart disease has dropped his blood pressure drop to 80/40 mmHg (MAP 53 mmHg).The surgical (below knee amputation) wound is oozing. The hemoglobin has dropped from 11 to 9 g/dL. His ECG shows new T wave inversion in the anterior leads What is the primary target of initial resuscitation in this scenario: Select one: a. elevate mean arterial pressure b. elevate pulse pressure c. elevate diastolic blood pressure d. elevate systolic pressure e. elevate hemoglobin level
C
120
A hypotensive patient has cold peripheries, heart rate of 120/min and JVP 0. The most likely cause of shock is: Select one: A. Sepsis B. Hypovolaemia C. Massive pulmonary embolus D. Anaphylaxis E. Myocardial infarction
B
121
A 70-year old man, who has history of COPD, presents with productive cough, fever and shortness of breath. He is given inhaled salbutamol and ipratropium. He can only open his eyes on painful stimulation, localise pain and produce some incomprehensible sound. You find lots of sputum sound on your physical examination.The following is his latest ABG result while on Hudson's mask with 6L/min flow of oxygen, pH 7.24 pCO2 7.5kPa (56mmHg) pO2 9.5kPa (71.3mmHg) HCO3: 25.6mmol/L What will be your next step(s) of management? Select one or more: a. Change to non-rebreathing mask with 15/min flow of oxgen b. IV theophylline c. Non-invasive ventilation d. IV Ceftriaxone e. Refer to ICU for intubation f. Add iv hydrocortisone
D,E,F
122
You are resuscitating a 70 year old patient with septic shock. He has received 2L Hartmann's solution in the past hour and has only passed 30 ml urine. On examination his BP is 90/40, HR 120/min, JVP 3 cm above sternal angle, SpO2 96% on oxygen 6l/min. His blood pressure rises to 100/40 on passive leg raising. He has received broad spectrum antibiotics for his presumed urinary tract infection. The next step in his resuscitation is: Select one: a. Continued fluid resuscitation with dextrose 5% b. Continued fluid resuscitation with Hartmann's solution c. Norepinephrine infusion d. Frusemide bolus e. Dopamine infusion
B
123
A patient has suddenly become unconscious in his bed, and you are the first doctor to arrive. The patient is not breathing. His pulse is present, and regular. You attempt to ventilate the patient after applying a triple manoeuvre. The chest fails to rise when the bag is squeezed. Your immediate airway management should be: Select one: a. modified jaw thrust b. intubation c. placement of an oropharyngeal airway d. Check and ensure adequate seal around the mask e. two-person technique to assist ventilation
D
124
A patient has suddenly become unconscious in his bed, and you are the first doctor to arrive. The patient is not breathing. His pulse is present, and regular. You attempt to ventilate the patient after applying a triple manoeuvre. The chest fails to rise when the bag is squeezed even although you have an adequate seal around the mask. Your immediate airway management should be: Select one: a. intubation b. placement of an oropharyngeal airway (partially correct) c. check you have an adequate head tilt and chin lift d. two-person technique to assist ventilation e. modified jaw thrust
C
125
Indicate in which order you would see the following patients A 72 year old woman who was admitted for a elective hemicolectomy who fell out of bed 30 minutes ago. The nurses tell you that her GCS is 3 and her left pupil is dilated A 72 year old woman with a history of idiopathic epilepsy who has just had a generalized convulsion which terminated spontaneously. She is being nursed in the left lateral position, has a patent airway, respiratory rate 20/min, SpO2 96% on air, capillary glucose 4 mmol/l. GCS 7. A 72 year old woman was admitted for a elective hemicolectomy tomorrow. She suddenly developed a left hemiparesis 1 hour ago. Her GCS is 15.
A,D,B,C
126
You are asked to review a patient who has been resuscitated from hypotensive shock. He is awaiting transfer to the ICU but it is currently full and he will not be able to be admitted to ICU for another 30 minutes. So far he has only received supportive treatment. He gives a history of fever, cough and shortness of breath progressing over 2 days. On examination he has bronchial breathing over the right lower zone. His heart rate is 120/min, BP 110/50 (120/50 after passive leg raising), JVP 1 cm above sternal angle, SpO2 94% on 6 l/min oxygen, urine output 30 ml since urinary catheter was inserted 30 mins ago. He is currently receiving dopamine 5 mcg/kg/min The management priority is: (choose 3, in descending order of priority) (P. 43) A. Further fluid resuscitation B. Give IV antibiotics C. Blood culture D. Send blood for complete blood count E. History taking, including allergy F. Reduce dopamine infusion rate
E,C,B History taking, including allergy Blood culture Give IV antibiotics
127
Which device delivers an inspired oxygen concentation close to 100%: Select one: a. Simple face mask with oxygen flow rate of 15L/min b. Reservoir oxygen mask with oxygen flow rate of 15L/min c. Bag valve mask resuscitator with reservoir bag and oxygen flow rate of 15L/min Correct (provided mask is held tightly to the face) d. 60% Venturi mask with oxygen flow rate of 30L/min (instead of usual 15L/min) e. Nasal cannulae with oxygen flow rate of 15L/min
C
128
You review a 76 year old patient who you have been resuscitating from septic shock due to urinary tract infection. He has had 2 L normal saline. He is confused and has the following haemodynamic findings: Heart rate 120/min (decreased from 140) BP 100/40 after passive leg raising (95/35 before passive leg raising) Warm peripheries Your next step in haemodynamic resuscitation is: Select one: a. Give another 1L normal saline over 30 mins b. Give epinephrine 0.1 mg IV bolus c. Start IV infusion of dobutamine (wrong: Is the MAP adequate?) d. Start IV infusion of dopamine e. No further haemodynamic resuscitation is necessary
D
129
A 64 year old, 80kg diabetic man presents with a severe urinary tract infection. He has a blood pressure of 82/43mmHg and a pulse rate of 123 beats/min. He is mildly confused, but his peripheries are warm. Urine output has been 120mL in the last 5h. He has been given 1.5L Hartman’s solution intravenously over the last hour and his JVP is now 5cm above the sternal angle. You have ordered a dopamine infusion and the nurse has started the infusion (in the ward standard concentration) at 5mL/h. What would be your initial target, against which you would titrate the dopamine infusion rate? (P. 56) Select one: a. A urine output of at least 1mL/kg/h b. A systolic blood pressure of at least 100mmHg c. A mean blood pressure of at least 65mmHg d. A decrease in heart rate to less than 100beats/minute e. A diastolic blood pressure of at least 60mmHg
C
130
A 65 years old lady was admitted for severe left loin pain, fever, chills and rigor. She was noted to have hypotension on presentation. Her BP is 105/65 HR 105/min after 1L of crystalloid given. Two sets of blood culture are taken and broad spectrum antibiotics are given. Her blood test shows WCC 13, Hb 9g/dL, Platelet 90 and INR 1.4. USG urinary system find left side pyo-nephrosis (renal abscess). What is your next step of management? (P. 93) Select one: a. Start dopamine infusion b. Discuss with your senior for drainage of pyo-nephrosis (renal abscess) c. Transfuse fresh frozen plasma d. Start noradrenaline infusion e. Transfuse platelet concentrate
B
131
You have witnessed a man suddenly collapse in the basement of your hospital. He is pulseless, but is gasping. There is no one nearby. The correct immediate management is: Select one: (P. 65) a. Immediately start mouth-to-mouth rescue breathing b. Leave the patient and search for an automated external defibrillator c. Call for help, but if no one is near, do not leave the patient to get help, and immediately start chest compressions d. The patient has gasping breathing efforts and so has not arrested and can be observed in the recovery position e. Call for help, but as there is nobody near, you find someone to raise the arrest call, and only then start chest compressions
E
132
You are managing a deeply unconscious patient (with no evidence of trauma) who is obstructing his airway despite a triple manoeuvre. The next step in his management is to: (P. 25) Select one: a. Use a two person technique to open the airway and squeeze the bag b. Insert a nasopharyngeal airway c. Perform a head-tilt, chin lift d. Perform a modified jaw thrust e. Insert an oropharyngeal airway
E
133
You have been asked to see four patients. What order would you see them in? 56 year old with severe crushing chest pain which started 20 minutes ago and has not been relieved by glyceryl trinitrate. 68 year old man who underwent a laparotomy today and is complaining of abdominal pain. SpO2 98% on 2l/min oxygen via nasal cannulae, respiratory rate 28/min, heart rate 98/min, BP 160/90. 72 year old woman with a urinary tract infection. The ward have just phoned to tell you her plasma potassium is 3.2 mmol/l. 47 year old admitted with right upper quadrant pain and jaundice. He is febrile and has been oliguric for the past hour.
A,D,B,C
134
You are called to see a 68 year old man with BP 120/90. His speech is confused, heart rate 130/min regular, capillary refill time 5 seconds, JVP 0 cm, BP after passive leg raising 125/85. Initial management should be: Select one: [similar questions] a. Give 1L Normal saline over 30 minutes b. Find and attach a pulse oximeter c. Give 100 ml Normal saline over 30 minutes d. Give 1L Normal saline as fast as possible e. Check airway
A
135
You are called to see a 68 year old man with BP 90/60. His speech is confused, heart rate 130/min regular, capillary refill time 5 seconds, JVP 0 cm, BP after passive leg raising 100/65. Initial management should be: Select one: [similar questions] a. Check airway b. Give 1L Normal saline over 30 minutes (wrong) c. Give 1L Normal saline as fast as possible d. Find and attach a pulse oximeter e. Give 100 ml Normal saline over 30 minutes
C
136
A 55 year old man patient was admitted for urinary tract infection with fever, heart rate of 110 beats/minute and blood pressure 85/45 mmHg, his urine output is only ~10ml in the one hour since admission. His potassium is 5 mmol/l, urea 15 mmol/l and creatinine 280 mcmol/l, pH 7.35 BE -4 . What would you do next in addition to fluid resuscitation? Select one: a. Arrange for urgent imaging of urinary system b. Perform culture and give antibiotics c. Give furosemide d. Give bicarbonate e. Arrange for renal replacement therapy
B
137
In which order would you see the following patients? A 53 year old man admitted for an elective cholecystectomy. The nurses ring you to tell you his potassium is 5.6 mmol/l 52 year old asthmatic patient who the nurses say is complaining of increasing shortness of breath. Her respiratory rate is 35/min and she is unable to speak in full sentences. Her PEFR is 80 l/min A 54 year old man who has been anuric for 4 hours
B,C,A
138
A patient presents with shortness of breath which came on while he was shopping for food. On examination: heart rate 113/min, BP 190/100, JVP + 9 cm, pan systolic murmur at apex, radiating to axilla, cold peripheries, bilateral crackles, SpO2 93% on 15 l/min oxygen. Immediate management should include; (P. 80) Select one or more: a. Refer to ICU for intubation and invasive ventilation b. Refer to cardiothoracic surgeon for emergency mitral valve surgery c. CT pulmonary angiogram d. Non invasive ventilation e. Chest X-ray
D,E A: (No. Try NIV first.) B: (No. Resuscitate first) D: (pulmonary oedema responds well to non invasive ventilation) E: (to confirm diagnosis) Explanation: Sympathetic response is common in pulmonary oedema, causing wide pulse pressure
139
A 55 years old man is admitted to the orthopaedic ward 4 hours ago because of cervical spine injury after fall from height. The nurse informs you this patient’s blood gas result. His blood gas shows PaCO2 10.7 kPa and PaO2 5.3 kPa while on room air. What is the most likely pathophysiology of his respiratory failure? Select one: a. All of the above b. Shunting c. Dead space ventilation d. Diffusion abnormality e. Hypoventilation
E
140
An 80 years old patient day 1 post-below-knee amputation was found to have a blood pressure drop to 80/40 mmHg (MAP 53 mmHg).The surgical wound is oozing. The hemoglobin has dropped from 10 to 8 g/dL. What is the primary target of initial resuscitation in this scenario: Select one: a. elevate diastolic blood pressure b. elevate pulse pressure c. elevate mean arterial pressure d. elevate hemoglobin level e. elevate systolic pressure
C
141
A 58 year old diabetic patient is admitted to medical ward for right foot pain, fever, chills and rigor. His right foot is erythematous and grossly swollen. His blood pressure is 110/65 and HR 110/min after 1L of saline given. X-Ray of his right foot is unremarkable. Two sets of blood culture is taken and broad spectrum antibiotics is started. What should be your next step? Select one: a. Give 500ml colloid full rate b. Give 500ml crystalloid full rate c. Start dopamine infusion d. Give dobutamine infusion e. Consult orthopaedic surgeon for assessment
E
142
A 55 year old obese lady who had a right total knee replacement 7 days ago develops right calf swelling for 2 days while rehabilitating in ward. Her haemoglobin level is 10.1g/dl this morning. She will have an USG Doppler examination of her lower limbs this afternoon, but develops shortness of breath and dizziness just after initiating her physiotherapy this morning. Her blood pressure is 80/40, pulse rate 110/min and SpO2 86% on room air. What would be your immediate response: (P. 54) Select one or more: a. Give gelofusine 500 ml b. Start dopamine infusion 5 mcg/min/kg c. Give high flow oxygen via a facemask d. Give normal saline 250 ml as fast as possible e. Give epinephrine 0.1 mg IV (Wrong)
C,D Explanation: Her immediate problems are shock and respiratory failure. She needs oxygen and treatment to reverse her respiratory failure. Based on the history of sudden onset of shortness of breath and dizziness and high risk for DVT, PE is the most likely cause. Therefore, give a moderate fluid challenge to try to reverse her hypotension.
143
You are managing a patient who says he has severe shortness of breath. His SpO2 is 88% on reservoir face mask 15 l/min oxygen. The ICU has been consulted and are on the way to assess the patient. In the meantime you should (select one or more): Select one or more: a. Apply a bag valve mask resuscitator with oxygen 15 l/min and allow the patient to breath spontaneously b. Insert a nasopharyngeal airway c. Ventilate the patient with a bag valve mask resuscitator with oxygen 15 l/min d. Insert an oral airway e. Perform a head tilt chin lift
A B:(wrong, He is speaking so is unlikely to have a major airway problem.)
144
A 52 year old man with a severe bacterial urinary infection has developed the following clinical features. Temperature 40C. BP 110/32 (mean 54mmHg), CVP 8 mmHg before fluid challenge and 14 mmHg after, urine output is 10 ml/hr, pH 7.34, bicarbonate 18 mmol/l, K+ 4.9 mmol/l, creatinine 370 µmol/l. The most appropriate initial treatment is: (P. 56) Select one: a. Dopamine infusion (fluid or vasopressor, fluid adequate, dopamine because more handy) b. Frusemide 100 mg IV c. Epinephrine 0.1 mg IV d. Fluid bolus 300 ml normal saline (wrong) e. Urgent renal replacement therapy
A
145
You have fluid resuscitated him with a 100 ml of normal saline and started him on a dopamine infusion. He should be treated with: [similar questions] (P. 77) Select one: a. aspirin, low molecular weight heparin, beta blocker, intravenous glycoprotein IIb/IIIa inhibitor and clopidogrel b. aspirin plus tissue plasminogen activator c. aspirin alone d. aspirin, enoxaparin, second anti-platelet agent and early angiography e. aspirin, low molecular weight heparin, beta blocker oral glycoprotein IIa/IIIb inhibitor and clopidogrel
D
146
You are asked to see a patient with COPD who has desaturated. He is not breathing. The appropriate method of improving his oxygenation is: Select one: a. Oxygen 6L/min via Hudson mask b. Bag-mask ventilation with 15 l/min oxygen c. Oxygen 15L/min via reservoir face mask d. Oxygen 4L/min via nasal cannulae e. Oxygen 2L/min via nasal cannulae
B
147
A 22 year old man was admitted to the orthopaedic ward two hour ago because of right thigh open fracture after a bicycle injury. He is now awaiting for an emergency operation. The nurse informs you that he is agitated, his heart rate is 120/min, SBP 100mmHg and respiratory rate 28/min and SpO2 96% on room air. His periphery is cold and no urine output was recorded after admission. What is your next management? Select one: a. Give intravenous midazolam as sedation b. Give oxygen through non re-breathing mask c. Blood culture and start antibiotics d. Prepare chest drain insertion e. Give warm saline through a large iv cannula
E
148
A 75 year old woman is admitted for sepsis due to liver abscess. She is transferred back to the medical ward from the X-ray department after percutaneous drainage of the abscess. The nurse reports to you that her blood pressure is 50/25 and pulse rate 120/min. When you assess her, she is drowsy, her periphery is warm. What will you do next? [similar questions] Select one or more: a. Call for help b. Start dobutamine 2µg/kg/min infusion c. Start nor epinephrine 0.05µg/kg/min infusion (wrong, she won’t survive long enough for you to set up an infusion) d. Give epinephrine 1mg intravenously e. Give epinephrine 0.1mg intravenously
A,E
149
A 75 year old woman is admitted for sepsis due to liver abscess. She is transferred back to the medical ward from the X-ray department after percutaneous drainage of the abscess. The nurse reports to you that her blood pressure is 60/25 and pulse rate 120/min. When you assess her, she is drowsy, her periphery is warm. After passive leg raising her BP rises to 90/40. What will you do next? [similar questions] Select one: a. Give 1l normal saline over 30 minutes b. Give 1l normal saline IV as fast as possible c. Start nor epinephrine 0.05µg/kg/min infusion d. Give epinephrine 0.1mg intravenously (wrong, she doesn’t have life threatening hypotension) e. Start dobutamine 2µg/kg/min infusion
B
150
A 78 year old man was admitted to the medical ward for 2 day history of fever, cough with purulent sputum and progressive shortness of breath. His blood pressure was 70/30 mmHg, heart rate 124 beats/minute, and the JVP visible 1 cm above the sternal angle. His SpO2 is 90 % on 6L/min of oxygen via a Hudson mask and his respiratory rate is 25 breaths/minute. There is bronchial breath sounds on the left lower zone of his lungs. His peripheries are warm and his urine output is 10ml/hour. What should be the next step of your management? Select one: a. Take blood for blood gas, complete blood count and renal function b. Give dopamine and titrate to MAP of 65mmHg or more c. Change to 15L/minute of oxygen via reservoir face mask d. Perform blood culture and then give dual an bio cs e. Give 1 liter normal saline, then reassess before giving further boluses
E Explanation: give fluid in pneumonia (p.44)
151
Look at the picture below which shows attempted bag mask ventilation in a patient who is apnoeic and unconscious but does not have a history of trauma. The chest is not rising with ventilation. What is wrong with the technique? [no picture] (P. 27) Select one or more: a. The bag is not being squeezed sufficiently b. The operator's fingers are not placed appropriately to keep the airway open c. The operator has not positioned the head appropriately d. The operator has not inserted an oropharyngeal airway e. The mask is not being held appropriately on the face
B,C,E
152
You are called to review a 60 year old man with liver abscess who had a percutaneous drainage of the abscess 1 hour ago. When you assess this patient, his heart rate is 130/min on cardiac monitoring and his blood pressure is un-recordable but his airway is patent and he is breathing. What would you do first: Select one: a. Give oxygen 15 l/min via a reservoir facemask b. Give normal saline 1L as fast as possible c. Give Gelofusine 500ml as fast as possible d. Give broad spectrum antibiotics e. Give epinephrine 0.1 mg IV
E Explanation: Whenever you assess a critically ill patient you should consider how much time you have and what is going to kill the patient first. In this case the blood pressure is un- recordable and the patient is about to die from severe shock in the next few minutes.
153
A 60 year old man was admitted for septic shock due to urinary tract infection. He is drowsy with warm periphery on assessment. There is no urine in his urinary bag after foley cath insertion 2 hours ago. His systolic blood pressure drops to 50mmhg despite IV crystalloid bolus of 1L. Your next action should be Select one: a. Give 500 ml crystalloid IV as fast as possible b. Give epinephrine 0.1mg intravenously c. Start epinephrine 0.05μg/kg/min infusion d. Start dobutamine 2μg/kg/min infusion e. Start norepinephrine 0.05μg/kg/min infusion
B
154
A 65 year old man with congestive heart failure was admitted for dyspnoea. On examination he was on 6L/min of facemask oxygen with SpO2 95% and BP 160/90. He had pitting oedema up to his knees and JVP at 4cm above clavicle. Chest examination showed bilateral crackles with mild use of accessory muscles. His urine output was 30mL/hour. How will you treat this patient? Select one: a. increase oxygen flow rate to 8L/min b. frusemide c. refer to renal team for dialysis d. dopamine infusion e. fluid restriction only
B
155
A 44-year-old patient with a perforated duodenal ulcer is day 4 post-operation. During the morning ward round, he was found to have a temperature of 39.5 C, have a blood pressure of 82/34 mmHg, a pulse rate of 126/min and a JVP just visible at the sternal angle. He was immediately given a bolus of 500mL Hartman’s solution over 20 minutes. After that, his blood pressure is 96/38 mmHg and JVP is 1cm above the sternal angle. His pulse rate is 110/min and he has warm peripheries on palpation. His urine output is 25ml/h in the last hour. The most appropriate treatment at this stage is: Select one: a. Repeat the fluid bolus using a volume of 500 mL Hartman’s solution. b. Repeat the fluid bolus using a volume of 50 mL Hartman’s solution. c. Give a bolus of epinephrine 0.1mg intravenously. d. Order an urgent ECG. e. Immediately start a dopamine or epinephrine infusion according to the standard ward protocol.
A
156
A 68-year-old woman presents with sudden onset shortness of breath. The blood pressure was 84/29mmHg, heart rate 124 beats per minute, and the JVP is 6 cm above the level of the sternal angle. She has received a bolus of 250 mL of normal saline. BP after fluid bolus 90/35 mm Hg. Her peripheries are cold, and capillary refill 4 seconds. Arterial oxygen saturation is 97% (she has a non-rebreathing reservoir mask with 15L/min oxygen flow) You call for help from the ICU, but in the meantime the best management is: Select one: a. Administer an intravenous epinephrine bolus of 0.1mg (wrong. Does the patient have life threatening hypotension?) b. Start a dopamine infusion c. Change oxygen mask to a 40% venturi-type device d. Begin a further intravenous fluid bolus of 250mL normal saline e. Start a Dobutamine infusion
D
157
A 65 year old smoker presents with sudden onset of shortness of breath. On examination he has a heart rate of 110/min, BP 170/100, JVP 8 cm above sternal angle, bilateral crackles. His arterial blood gas on oxygen 15L/min via reservoir facemask shows: pH 6.9, PaCO2 7.2 kPa (54 mmHg), PaO2 10 kPa (75 mmHg), HCO3- 10 mmol/L. The most likely diagnosis is: (P.124) Select one: a. Diabetic ketoacidosis b. Hepatic and renal failure c. Acute exacerbation of chronic obstructive pulmonary disease d. Severe acute pulmonary oedema (metabolic and respiratory acidosis must consider APO) e. Pneumonia
D
158
You see a 24 year old 50 kg patient with a history of asthma who tells you that she suddenly became more breathless on her way to work. On examination her respiratory rate is 24/min, chest expansion decreased on the right, no wheeze, breath sounds slightly decreased on right, PEFR 380 l/min, SpO2 95% on 6l/min oxygen via a simple facemask. She is tachycardic but otherwise haemodynamically stable. Your next step should be: Select one: a. Obtain an arterial blood gas b. Start IV hydrocortisone c. Give salbutamol and ipratropium inhalers d. Request an urgent chest X-ray e. Refer for ICU admission
D